Help please it’s emergency: I don’t understand how to do number 7

Help Please Its Emergency: I Dont Understand How To Do Number 7

Answers

Answer 1

With a greater mean value , we can conclude that the sixth period class test was better than the second period .

Calculating the mean of each class

Second period class:

Mean = (55+70+6*75+6*80+2*85+3*90+95)/20

Mean = 1590/20 = 79.5

Sixth period class:

Mean = (65+3*75+5*80+6*85+3*90+2*95)/20

Mean = 1660/20 = 83

Therefore, From the mean values , we can infer that students performed better in test for the sixth period class than the second .

Learn more on mean : https://brainly.com/question/20118982

#SPJ1


Related Questions

Assignment 2 Useful summation formulas and rules Σ 1≤i≤n

1=1+1+…+1=n−l+1 In particular, Σ 1≤i≤n

1=n−1+1=n∈Θ(n) Σ 1≤i≤n

i=1+2+…+n=n(n+1)/2≈n 2
/2∈Θ(n 2
) Σ 1≤k,n

i 2
=1 2
+2 2
+…+n 2
=n(n+1)(2n+1)/6≈n 3/3
∈Θ(n 3
) 1 k
+2 k
+3 k
+⋯+n k
≤n k
+n k
+n k
+⋯+n k
=n k+1
∈Θ(n k+1
) Σ 0≤i≤n

a i
=1+a+…+a n
=(a n+1
−1)/(a−1) for any a

=1 In particular, Σ 0<5n

2 i
=2 0
+2 1
+…+2 n
=2 n+1
−1∈Θ(2 n
) Σ(a i

±b i

)=Σa i

±Σb i

;Σca i

=cΣa i

;Σ l≤1≤n

a i

=Σ l≤i≤m

a i

+Σ m+1≤i≤n

a i

By the use of the above summation formula calculate the exact number of basic operation of the following examples and the recurrence relation and their backward substitution and then deduce the theta and the Big O of the following functions. Recursive definition of n!:F(n)=F(n−1)∗n for n≥1 and F(0)=1 ecurrence for number of moves: M(n)=M(n−1)+1+M(n−1) ALGORITHM BinRec(n) //Input: A positive decimal integer n //Output: The number of binary digits in n 's binary representation if n=1 return 1 else return BinRec(⌊n/2⌋)+1

Answers

The exact number of basic operations, recurrence relations, and the complexity analysis (Theta and Big O) for the given examples are as follows: Recursive definition of n!, Recurrence for the number of moves, Algorithm BinRec(n).

Let's go over each one to determine the exact number of basic operations and the recurrence relation for the given examples:

Definition of n! in a recursive way:

Operation basics: Relation of recurrence and multiplication: Backward substitution: F(n) = F(n-1) * n

Deduction of Theta and Big O: F(n) = F(n-1) * n F(n-1) = F(n-2) * (n-1)... F(2) = F(1) * 2 F(1) = F(0) * 1

Each recursive call performs a multiplication, with n calls total.

As a result, O(n) is the Big O and Theta(n) is the number of basic operations.

For the number of moves, recurrence:

Operation basics: Relation of addition and recurrence: M(n) is equal to M(n-1) plus 1 and M(n-1).

Deduction of Theta and Big O: M(n) = M(n-1) + 1 + M(n-1) M(n-1) = M(n-1) + 1 + M(n-2)... M(2) = M(1) + 1 + M(1) M(1) = M(0) + 1 + M(0)

Each recursive call adds to the total number of calls, which is 2n - 1.

As a result, O(2n) is the Big O and Theta(2n) is the number of basic operations.

The BinRec(n) algorithm:

Operation basics: Division and addition (floor) Relation to recurrence: Backward substitution: BinRec(n) = BinRec(floor(n/2)) + 1.

Theta and Big O can be deduced as follows: BinRec(n) = BinRec(floor(n/2)) + 1 BinRec(floor(n/2)) = BinRec(floor(floor(n/2)/2)) + 1

The quantity of recursive calls is log(n) (base 2), and each call plays out an expansion and a division.

As a result, O(log n) is the Big O and Theta(log n) is the number of basic operations.

For the given examples, the exact number of basic operations, recurrence relations, and complexity analysis (Theta and Big O) is as follows:

Definition of n! in a recursive way:

Basic procedures: Relation of recurrence in theta(n): Theta: F(n) = F(n-1) * n Big O: Theta(n): O(n) Repeatability for the number of moves:

Basic procedures: Relation of recurrence in theta(2n): Theta: M(n) = M(n-1) + 1 + M(n-1) Big O: Theta(2n) Algorithm BinRec(n): O(n)

Basic procedures: Relation of recurrence: theta(log(n)). BinRec(n) is equal to BinRec(floor(n/2)) plus one Theta: Big O: Theta(log(n)) O(log(n)) Please note that the preceding analysis assumes constant time complexity for the fundamental operations of addition, division, and multiplication.

To know more about Recurrence relations, visit

brainly.com/question/4082048

#SPJ11

Solve the initial Valve Problem. dx/dy=(y/x+x/y),y(1)=−4

Answers

To solve the initial value problem (IVP) dx/dy = (y/x) + (x/y) with the initial condition y(1) = -4, we can use a change of variables. Let's define a new variable u = x/y. Then we have x = uy.

Differentiating both sides with respect to y using the chain rule, we get:

dx/dy = d(uy)/dy = u(dy/dy) + y(du/dy) = u + y(du/dy).

Substituting this back into the original equation, we have:

u + y(du/dy) = (y/x) + (x/y).

Since x = uy, we can rewrite the equation as:

u + y(du/dy) = (y/(uy)) + (uy)/y.

Simplifying further, we have:

u + y(du/dy) = 1/u + u.

Now, we can separate the variables by moving all the terms involving u to one side and all the terms involving y to the other side:

(du/dy) = (1/u + u - u)/y.

Simplifying this expression, we get:

(du/dy) = (1/u)/y.

Now, we can integrate both sides with respect to y:

∫ (du/dy) dy = ∫ (1/u)/y dy.

Integrating, we have:

u = ln(|y|) + C,

where C is the constant of integration.

Substituting back u = x/y, we have:

x/y = ln(|y|) + C.

Multiplying both sides by y, we get:

x = y ln(|y|) + Cy.

Now, we can use the initial condition y(1) = -4 to solve for the constant C:

-4 = ln(|1|) + C.

Since ln(|1|) = 0, we have:

-4 = C.

Therefore, the particular solution to the IVP is given by:

x = y ln(|y|) - 4y.

This is the solution to the initial value problem dx/dy = (y/x) + (x/y), y(1) = -4.

Learn more about initial value here:

https://brainly.com/question/17613893

#SPJ11

Find the equation of the plane that is parallel to the vectors ⟨1,0,2⟩ and ⟨0,2,1⟩, passing through the point (4,0,−4). The equation of the plane is (Type an equation using x,y, and z as the variables.)

Answers

To find the equation of the plane parallel to the vectors ⟨1,0,2⟩ and ⟨0,2,1⟩ and passing through the point (4,0,−4), we can use the formula for the equation of a plane.

The equation of a plane is given by Ax + By + Cz = D, where A, B, C are the coefficients of the normal vector to the plane, and (x, y, z) are the coordinates of a point on the plane.

Since the plane is parallel to the given vectors, the normal vector of the plane can be found by taking the cross product of the two given vectors. Let's denote the normal vector as ⟨A, B, C⟩.

⟨A, B, C⟩ = ⟨1, 0, 2⟩ × ⟨0, 2, 1⟩

= (01 - 20)i + (12 - 01)j + (10 - 22)k

= 0i + 2j - 4k

= ⟨0, 2, -4⟩

Now, we have the normal vector ⟨A, B, C⟩ = ⟨0, 2, -4⟩ and a point on the plane (4, 0, -4). Plugging these values into the equation of a plane, we get:

0x + 2y - 4z = D

To find the value of D, we substitute the coordinates of the given point (4, 0, -4):

04 + 20 - 4*(-4) = D

0 + 0 + 16 = D

D = 16

Therefore, the equation of the plane is:

0x + 2y - 4z = 16

Simplifying further, we get:

2y - 4z = 16

This is the equation of the plane parallel to the given vectors and passing through the point (4, 0, -4).

Learn more about equation here: brainly.com/question/30130739

#SPJ11

What is the result of this numerical calculation using the correct
number of significant figures? (55".0100 + 37.0".0156 +
48.15*1.27E-3) / (0.02000 * 78.12 )

Answers

The result of the numerical calculation, rounded to the appropriate number of significant figures, is approximately 82.60. This takes into account the significant figures of the values and ensures the proper precision of the final result.

To perform the numerical calculation with the correct number of significant figures, we will use the values and round the final result to the appropriate number of significant figures.

(55.0100 + 37.0 + 48.15 * 1.27E-3) / (0.02000 * 78.12)

= (92.0100 + 37.0 + 0.061405) / (0.02000 * 78.12)

= 129.071405 / 1.5624

= 82.603579

Rounded to the correct number of significant figures, the result of the calculation is approximately 82.60.

To know more about numerical calculation refer here:

https://brainly.com/question/32839846#

#SPJ11

The movement of the progress bar may be uneven because questions can be worth more or less (including zero ) depent What are the exponent and coefficient of the expression -5b ?

Answers

The exponent and coefficient of the expression -5b are 1 and -5, respectively.

To find the exponent and coefficient of the expression, follow these steps:

An exponent is a mathematical operation that shows how many times a number or expression is multiplied by itself. So, for the expression -5b, the exponent is 1 as b is multiplied by itself only once. A coefficient is a numerical value that appears before a variable or a term in an algebraic expression. So, for the expression -5b, the coefficient is -5 because it is the number that appear before the variable b.

Therefore, the exponent is 1 and the coefficient is -5.

Learn more about exponent:

brainly.com/question/11975096

#SPJ11

Find a and b such that the following function is a cdf: G(x)= ⎩



0
a(1+cos(b(x+1))
1

x≤0
0 x>1

Answers

The values of a and b that make the given function a CDF are a = 0 and b = 1.

To find a and b such that the given function is a CDF, we need to make sure of two things:

i) F(x) is non-negative for all x, and

ii) F(x) is bounded by 0 and 1. (i.e., 0 ≤ F(x) ≤ 1)

First, we will calculate F(x). We are given G(x), which is the CDF of the random variable X.

So, to find the PDF, we need to differentiate G(x) with respect to x.  

That is, F(x) = G'(x) where

G'(x) = d/dx

G(x) = d/dx [a(1 + cos[b(x + 1)])] for x ≤ 0

G'(x) = d/dx G(x) = 0 for x > 1

Note that G(x) is a constant function for x > 1 as G(x) does not change for x > 1. For x ≤ 0, we can differentiate G(x) using chain rule.

We get G'(x) = d/dx [a(1 + cos[b(x + 1)])] = -a.b.sin[b(x + 1)]

Note that the range of cos function is [-1, 1].

Therefore, 0 ≤ G(x) ≤ 2a for all x ≤ 0.So, we have F(x) = G'(x) = -a.b.sin[b(x + 1)] for x ≤ 0 and F(x) = 0 for x > 1.We need to choose a and b such that F(x) is non-negative for all x and is bounded by 0 and 1.

Therefore, we need to choose a and b such that

i) F(x) ≥ 0 for all x, andii) 0 ≤ F(x) ≤ 1 for all x.To ensure that F(x) is non-negative for all x, we need to choose a and b such that sin[b(x + 1)] ≤ 0 for all x ≤ 0.

This is possible only if b is positive (since sin function is negative in the third quadrant).

Therefore, we choose b > 0.

To ensure that F(x) is bounded by 0 and 1, we need to choose a and b such that maximum value of F(x) is 1 and minimum value of F(x) is 0.

The maximum value of F(x) is 1 when x = 0. Therefore, we choose a.b.sin[b(0 + 1)] = a.b.sin(b) = 1. (This choice ensures that F(0) = 1).

To ensure that minimum value of F(x) is 0, we need to choose a such that minimum value of F(x) is 0. This happens when x = -1/b.

Therefore, we need to choose a such that F(-1/b) = -a.b.sin(0) = 0. This gives a = 0.The choice of a = 0 and b = 1 will make the given function a CDF. Therefore, the required values of a and b are a = 0 and b = 1.

We need to find a and b such that the given function G(x) = {0, x > 1, a(1 + cos[b(x + 1)]), x ≤ 0} is a CDF.To do this, we need to calculate the PDF of G(x) and check whether it is non-negative and bounded by 0 and 1.We know that PDF = G'(x), where G'(x) is the derivative of G(x).Therefore, F(x) = G'(x) = d/dx [a(1 + cos[b(x + 1)])] = -a.b.sin[b(x + 1)] for x ≤ 0F(x) = G'(x) = 0 for x > 1We need to choose a and b such that F(x) is non-negative and bounded by 0 and 1.To ensure that F(x) is non-negative, we need to choose b > 0.To ensure that F(x) is bounded by 0 and 1, we need to choose a such that F(-1/b) = 0 and a.b.sin[b] = 1. This gives a = 0 and b = 1.

Therefore, the values of a and b that make the given function a CDF are a = 0 and b = 1.

To know more about differentiate visit:

brainly.com/question/24062595

#SPJ11

Gentamycin 240 mg is ordered to be given q6h. what is the volume
needed for a 24 hour period if the concentration in stock is
40mg/ml?

Answers

For a 24-hour period, with Gentamycin 240 mg ordered q6h, the volume needed depends on the infusion rate.

To calculate the volume needed for a 24-hour period, we need to consider the dosing frequency and concentration of the stock solution.

Given that Gentamycin 240 mg is ordered q6h (every 6 hours), we can determine the total dosage required for a 24-hour period by multiplying the dosage per dose (240 mg) by the number of doses in 24 hours (24/6 = 4 doses).

Total dosage needed = 240 mg/dose * 4 doses = 960 mg

To find the volume needed, we divide the total dosage by the concentration of the stock solution. In this case, the concentration is 40 mg/ml.

Volume needed = Total dosage / Concentration = 960 mg / 40 mg/ml = 24 ml

Therefore, the volume needed for a 24-hour period, considering the given dosage and concentration, is 24 ml.

To learn more about “volume ” refer to the https://brainly.com/question/14197390

#SPJ11

the total revenue, r, for selling q units of a product is given by r=350q+55q^(2)-q^(3). Find the marginal revenue for selling 20 units."

Answers

Marginal revenue is the amount by which the revenue increases when the number of units sold is increased by one. The marginal revenue function is the derivative of the total revenue function.

[tex]`r'(20) = 350 + 110(20) - 3(20^2) = 350 + 2200 - 1200 = 1350`[/tex]

Hence, we need to differentiate the given revenue function to obtain the marginal revenue function. Marginal Revenue function can be derived from Total Revenue function.

`[tex]r = 350q + 55q^2 – q^3`[/tex]

[tex]`r' = 350 + 110q - 3q^2[/tex]`

[tex]`r'(20) = 350 + 110(20) - 3(20^2) = 350 + 2200 - 1200 = 1350`[/tex]

The marginal revenue for selling 20 units is 1350. The answer is verified to be correct.

To know more about Marginal visit:

https://brainly.com/question/28481234

#SPJ11

Parvati wants to donate enough money to Camosun College to fund an ongoing annual bursary of $1,500 to a deserving finance student. How much must she donate today in order for the first payment to to be given out right awav? Assume an interest rate of i 1

=4%. Camosun College has just received a donation of $100,000. The donor has stipulated that the funds should be used to fund an ongoing annual bursary of $4,750 with the first payment given out in one year. What is the minimum amount of interest (j 1

) that the funds must earn in order to make the bursary wark? Express your answer as a percent to 2 decimal places but don't include the % sign.

Answers

Parvati wants to donate enough money to Camosun College

a) Parvati needs to donate $1500 today to fund an annual bursary of $1500

b) The funds must earn a minimum interest rate of 4.75% to sustain an annual bursary

a) To calculate the amount Parvati needs to donate today, we can use the present value formula for an annuity:

PV = PMT / (1 + r)^n

Where PV is the present value, PMT is the annual payment, r is the interest rate, and n is the number of years.

In this case, Parvati wants to fund an ongoing annual bursary of $1,500 with the first payment given out immediately. The interest rate is 4%.

Calculating the present value:

PV = 1500 / (1 + 0.04)^0

PV = $1500

Therefore, Parvati must donate $1500 today to fund the ongoing annual bursary.

b) To determine the minimum amount of interest the funds must earn, we can use the present value formula for an annuity:

PV = PMT / (1 + r)^n

In this case, the donation is $100,000, and the annual payment for the bursary is $4,750 with the first payment given out in one year. We need to find the interest rate, which is represented as j.

Using the formula and rearranging for the interest rate:

j = [(PMT / PV)^(1/n) - 1] * 100

j = [(4750 / 100000)^(1/1) - 1] * 100

j ≈ 4.75%

Therefore, the minimum amount of interest the funds must earn to make the bursary work is 4.75%.

To learn more about interest rate visit:

https://brainly.com/question/29451175

#SPJ11

Part of the graph of the function f(x) = (x + 4)(x-6) is
shown below.
Which statements about the function are true? Select
two options.
The vertex of the function is at (1,-25).
The vertex of the function is at (1,-24).
The graph is increasing only on the interval -4< x <
6.
The graph is positive only on one interval, where x <
-4.
The graph is negative on the entire interval
-4

Answers

The statements that are true about the function are: The vertex of the function is at (1,-25), and the graph is negative on the entire interval -4 < x < 6.

1. The vertex of the function is at (1,-25): To determine the vertex of the function, we need to find the x-coordinate by using the formula x = -b/2a, where a and b are the coefficients of the quadratic function in the form of [tex]ax^2[/tex] + bx + c. In this case, the function is f(x) = (x + 4)(x - 6), so a = 1 and b = -2. Plugging these values into the formula, we get x = -(-2)/(2*1) = 1. To find the y-coordinate, we substitute the x-coordinate into the function: f(1) = (1 + 4)(1 - 6) = (-3)(-5) = 15. Therefore, the vertex of the function is (1,-25).

2. The graph is negative on the entire interval -4 < x < 6: To determine the sign of the graph, we can look at the factors of the quadratic function. Since both factors, (x + 4) and (x - 6), are multiplied together, the product will be negative if and only if one of the factors is negative and the other is positive. In the given interval, -4 < x < 6, both factors are negative because x is less than -4.

Therefore, the graph is negative on the entire interval -4 < x < 6.

The other statements are not true because the vertex of the function is at (1,-25) and not (1,-24), and the graph is negative on the entire interval -4 < x < 6 and not just on one interval where x < -4.

For more such questions on vertex, click on:

https://brainly.com/question/1217219

#SPJ8

Distance Two cyclists leave from an intersection at the same time. One travels due north at a speed of 15 miles per hour, and the other travels due east at a speed of 20 miles per hour. How long until the distance between the two cyclists is 75 mile

Answers

To solve this problem, we can use the Pythagorean theorem to find the distance between the two cyclists at any given time. Let's assume the time it takes for the distance between the two cyclists to be 75 miles is "t" hours.

The distance traveled by the cyclist traveling north is given by the formula: distance = speed × time.

Therefore, the distance traveled by the northbound cyclist after time "t" is 15t miles.

Similarly, the distance traveled by the cyclist traveling east is distance = speed × time.

So, the distance traveled by the eastbound cyclist after time "t" is 20t miles.

According to the Pythagorean theorem, the distance between the two cyclists is given by the square root of the sum of the squares of their respective distances traveled:

distance = sqrt((distance north)^2 + (distance east)^2)

Using the distances we found earlier, we can substitute them into the formula:

75 = sqrt((15t)^2 + (20t)^2)

Now, let's solve for "t" by squaring both sides of the equation:

5625 = (15t)^2 + (20t)^2

5625 = 225t^2 + 400t^2

5625 = 625t^2

t^2 = 5625 / 625

t^2 = 9

t = sqrt(9)

t = 3

Therefore, it will take 3 hours for the distance between the two cyclists to be 75 miles.

To learn more about Pythagorean theorem:https://brainly.com/question/343682

#SPJ11

write equation of a line passes through the point (1,-7) and has a slope of -9

Answers

The equation of a line that passes through the point (1, -7) and has a slope of -9 is y = -9x + 2

To find the equation of the line, follow these steps:

We can use the point-slope form of the equation of a line. The point-slope form is given by: y - y₁= m(x - x₁), where (x1, y1) is the point the line passes through and m is the slope of the line.Substituting the values of m= -9, x₁= 1 and y₁= -7, we get y - (-7) = -9(x - 1).Simplifying this equation: y + 7 = -9x + 9 ⇒y = -9x + 2.

Learn more about equation of line:

brainly.com/question/18831322

#SPJ11

Justin has $1200 in his savings account after the first month. The savings account pays no interest. He deposits an additional $60 each month thereafter. Which function (s) model the scenario?

Answers

Since the savings account pays no interest, we only need to use the linear function given above to model the scenario.

Given that Justin has $1200 in his savings account after the first month and deposits an additional $60 each month thereafter. We have to determine which function (s) model the scenario.The initial amount in Justin's account after the first month is $1200.

Depositing an additional $60 each month thereafter means that Justin's savings account increases by $60 every month.Therefore, the amount in Justin's account after n months is given by:

$$\text{Amount after n months} = 1200 + 60n$$

This is a linear function with a slope of 60, indicating that the amount in Justin's account increases by $60 every month.If the savings account had an interest rate, we would need to use a different function to model the scenario.

For example, if the account had a fixed annual interest rate, the amount in Justin's account after n years would be given by the compound interest formula:

$$\text{Amount after n years} = 1200(1+r)^n$$

where r is the annual interest rate as a decimal and n is the number of years.

However, since the savings account pays no interest, we only need to use the linear function given above to model the scenario.

For more such questions on linear function, click on:

https://brainly.com/question/2248255

#SPJ8

If 13x = 1989 ,then find the value of 7x.​

Answers

Answer:

1071

Step-by-step explanation:

1989÷13=153

so x=153

153×7=1071

so 7x=1071

Answer:

1,071

Explanation:

If 13x = 1,989, then I can find x by dividing 1,989 by 13:

[tex]\sf{13x=1,989}[/tex]

[tex]\sf{x=153}[/tex]

Multiply 153 by 7:

[tex]\sf{7\times153=1,071}[/tex]

Hence, the value of 7x is 1,071.

You put $422 per month in an investment plan that pays an APR of 3%. How much money will you have after 25 years? Compare this amount to the total amount of deposits made over the time period.

Answers

The total amount of money that will be available after 25 years is $191,727.98 and the total amount of deposits made over the time period is much less than the amount of money that will be available after 25 years.

Given that you put $422 per month in an investment plan that pays an APR of 3%.

We need to calculate how much money you will have after 25 years and compare this amount to the total amount of deposits made over the time period.

To find out the total amount of money that will be available after 25 years, we will use the formula for future value of an annuity.

FV = PMT * (((1 + r)n - 1) / r)

where,FV is the future value of annuity PMT is the payment per period n is the interest rate per period n is the total number of periodsIn this case,

PMT = $422r = 3% / 12 (monthly rate) = 0.25%n = 25 years * 12 months/year = 300 months.

Now, let's substitute the values in the formula,

FV = $422 * (((1 + 0.03/12)300 - 1) / (0.03/12))= $422 * (1.1378 / 0.0025)= $191,727.98.

Therefore, the total amount of money that will be available after 25 years is $191,727.98.

Now, let's calculate the total amount of deposits made over the time period.

Total deposits = PMT * n= $422 * 300= $126,600.

Comparing the two amounts, we can see that the total amount of deposits made over the time period is much less than the amount of money that will be available after 25 years.Therefore,investing in an annuity with a 3% APR is a good investment option.


To know more about amount click here:

https://brainly.com/question/32453941

#SPJ11

A piece of cheese is shaped like a triangle. It has a height of 4. 5 inches and a base that is 3. 25 inches long. If 1 inch = 2. 54 centimeters, find the area of the cheese in square centimeters. Round the answer to the nearest square centimeter. 19 cm2

Answers

Rounding this to the nearest square centimeter, the area of the cheese is approximately 47 cm².

To find the area of the cheese in square centimeters, we need to convert the given measurements from inches to centimeters and then calculate the area.

The height of the cheese is given as 4.5 inches. To convert this to centimeters, we multiply by the conversion factor:

4.5 inches * 2.54 cm/inch = 11.43 cm (rounded to two decimal places)

The base of the cheese is given as 3.25 inches. Converting this to centimeters:

3.25 inches * 2.54 cm/inch = 8.255 cm (rounded to three decimal places)

Now, we can calculate the area of the triangle using the formula:

Area = (1/2) * base * height

Area = (1/2) * 8.255 cm * 11.43 cm

Area ≈ 47.206 cm² (rounded to three decimal places)

Rounding this to the nearest square centimeter, the area of the cheese is approximately 47 cm².

It's important to note that the given answer of 19 cm² does not match the calculated result. Please double-check the calculations or provide further clarification if needed.

Learn more about  area  from

https://brainly.com/question/25292087

#SPJ11

Suppose that y is a solution to a first-order, d-dimensional, nonautonomous ODE dy/dt = f(t, y). (So a solution y = (y1,...,yd) can be thought of as a map R→ R^d, and f: RxR^d→ R^d.) Write a first- order, (d+1)-dimensional, autonomous ODE that is solved by w(t) = (t, y(t)). That is, t→ w(t) is a map from R→ R^d+1 (whose first component is t and whose last d components are given by the components of y), and I am asking you to find a function F: R^d+1 → R^d+1 such that dw/dt= F(w). (Hint: you know that dy/dt = f(t, y), and you also know what dt/dt is, so you can write down all of the components of dw/dt; this will become F(w). If the notation is confusing, start with the case when d = 1.) The upshot of this problem is that any non-autonomous ODE can be turned into an autonomous ODE, at the cost of increasing the dimension.

Answers

the first-order, (d+1)-dimensional, autonomous ODE solved by [tex]\(w(t) = (t, y(t))\) is \(\frac{dw}{dt} = F(w) = \left(1, f(w_1, w_2, ..., w_{d+1})\right)\).[/tex]

To find a first-order, (d+1)-dimensional, autonomous ODE that is solved by [tex]\(w(t) = (t, y(t))\)[/tex], we can write down the components of [tex]\(\frac{dw}{dt}\).[/tex]

Since[tex]\(w(t) = (t, y(t))\)[/tex], we have \(w = (w_1, w_2, ..., w_{d+1})\) where[tex]\(w_1 = t\) and \(w_2, w_3, ..., w_{d+1}\) are the components of \(y\).[/tex]

Now, let's consider the derivative of \(w\) with respect to \(t\):

[tex]\(\frac{dw}{dt} = \left(\frac{dw_1}{dt}, \frac{dw_2}{dt}, ..., \frac{dw_{d+1}}{dt}\right)\)[/tex]

We know that[tex]\(\frac{dy}{dt} = f(t, y)\), so \(\frac{dw_2}{dt} = f(t, y_1, y_2, ..., y_d)\) and similarly, \(\frac{dw_3}{dt} = f(t, y_1, y_2, ..., y_d)\), and so on, up to \(\frac{dw_{d+1}}{dt} = f(t, y_1, y_2, ..., y_d)\).[/tex]

Also, we have [tex]\(\frac{dw_1}{dt} = 1\), since \(w_1 = t\) and \(\frac{dt}{dt} = 1\)[/tex].

Therefore, the components of [tex]\(\frac{dw}{dt}\)[/tex]are given by:

[tex]\(\frac{dw_1}{dt} = 1\),\\\(\frac{dw_2}{dt} = f(t, y_1, y_2, ..., y_d)\),\\\(\frac{dw_3}{dt} = f(t, y_1, y_2, ..., y_d)\),\\...\(\frac{dw_{d+1}}{dt} = f(t, y_1, y_2, ..., y_d)\).\\[/tex]

Hence, the function \(F(w)\) that satisfies [tex]\(\frac{dw}{dt} = F(w)\) is:\(F(w) = \left(1, f(w_1, w_2, ..., w_{d+1})\right)\).[/tex]

[tex]\(w(t) = (t, y(t))\) is \(\frac{dw}{dt} = F(w) = \left(1, f(w_1, w_2, ..., w_{d+1})\right)\).[/tex]

Learn more about dimensional here :-

https://brainly.com/question/14481294

#SPJ11

Q5... Lids has obtained 23.75% of the
cap market in Ontario. If Lids sold 2600 caps last month, how many
caps were sold in Ontario in total last month? Round up the final
answer. (1 mark)

Answers

The total number of caps sold in Ontario last month is approximately 10948 caps (rounded up).

Given that Lids has obtained 23.75% of the cap market in Ontario and it sold 2600 caps last month. Let us calculate the total caps sold in Ontario last month as follows:

Let the total caps sold in Ontario be x capsLids has obtained 23.75% of the cap market in Ontario which means the percentage of the market Lids has not covered is (100 - 23.75)% = 76.25%.

The 76.25% of the cap market is represented as 76.25/100, hence, the caps sold in the market not covered by Lids is:

76.25/100 × x = 0.7625 x

The total number of caps sold in Ontario is equal to the sum of the number of caps sold by Lids and the number of caps sold in the market not covered by Lids, that is:

x = 2600 + 0.7625 x

Simplifying the equation by subtracting 0.7625x from both sides, we get;0.2375x = 2600

Dividing both sides by 0.2375, we obtain:

x = 2600 / 0.2375x

= 10947.37 ≈ 10948

Therefore, the total number of caps sold in Ontario last month is approximately 10948 caps (rounded up).Answer: 10948

To know more about percentage visit:

https://brainly.com/question/32197511

#SPJ11

Perform the indicated operation and simplify.
7/(x-4) - 2 / (4-x)
a. -1
b.5/X+4
c. 9/X-4
d.11/(x-4)

Answers

The simplified expression after performing the indicated operation is 9/(x - 4) (option c).

To simplify the expression (7/(x - 4)) - (2/(4 - x), we need to combine the two fractions into a single fraction with a common denominator.

The denominators are (x - 4) and (4 - x), which are essentially the same but with opposite signs. So we can rewrite the expression as 7/(x - 4) - 2/(-1)(x - 4).

Now, we can combine the fractions by finding a common denominator, which in this case is (x - 4). So the expression becomes (7 - 2(-1))/(x - 4).

Simplifying further, we have (7 + 2)/(x - 4) = 9/(x - 4).

Therefore, the simplified expression after performing the indicated operation is 9/(x - 4) (option c).

To learn more about fractions  click here

brainly.com/question/10354322

#SPJ11

Consider the following hypothesis statement using α=0.01 and data from two independent samples. Assume the population variances are equal and the populations are normally distributed. Complete parts a and b. H 0

:μ 1

−μ 2

≤8
H 1

:μ 1

−μ 2

>8

x
ˉ
1

=65.3
s 1

=18.5
n 1

=18

x
ˉ
2

=54.5
s 2

=17.8
n 2

=22

a. Calculate the appropriate test statistic and interpret the result. The test statistic is (Round to two decimal places as needed.) The critical value(s) is(are) (Round to two decimal places as needed. Use a comma to separate answers as needed.)

Answers

The given hypothesis statement isH 0: μ1 − μ2 ≤ 8H 1: μ1 − μ2 > 8The level of significance α is 0.01.

Assuming equal population variances and the normality of the populations, the test statistic for the hypothesis test is given by Z=(x1 − x2 − δ)/SE(x1 − x2), whereδ = 8x1 = 65.3, s1 = 18.5, and n1 = 18x2 = 54.5, s2 = 17.8, and n2 = 22The formula for the standard error of the difference between means is given by

SE(x1 − x2) =sqrt[(s1^2/n1)+(s2^2/n2)]

Here,

SE(x1 − x2) =sqrt[(18.5^2/18)+(17.8^2/22)] = 4.8862

Therefore,

Z = [65.3 - 54.5 - 8] / 4.8862= 0.6719

The appropriate test statistic is 0.67.Critical value:The critical value can be obtained from the z-table or calculated using the formula.z = (x - μ) / σ, where x is the value, μ is the mean and σ is the standard deviation.At 0.01 level of significance and the right-tailed test, the critical value is 2.33.The calculated test statistic (0.67) is less than the critical value (2.33).Conclusion:Since the calculated test statistic value is less than the critical value, we fail to reject the null hypothesis. Therefore, there is not enough evidence to support the alternative hypothesis at a 0.01 level of significance. Thus, we can conclude that there is insufficient evidence to indicate that the population mean difference is greater than 8. Hence, the null hypothesis is retained. The hypothesis test is done with level of significance α as 0.01. Given that the population variances are equal and the population distributions are normal. The null and alternative hypothesis can be stated as

H 0: μ1 − μ2 ≤ 8 and H 1: μ1 − μ2 > 8.

The formula to calculate the test statistic for this hypothesis test when the population variances are equal is given by Z=(x1 − x2 − δ)/SE(x1 − x2),

where δ = 8, x1 is the sample mean of the first sample, x2 is the sample mean of the second sample, and SE(x1 − x2) is the standard error of the difference between the sample means.The values given are x1 = 65.3, s1 = 18.5, n1 = 18, x2 = 54.5, s2 = 17.8, and n2 = 22The standard error of the difference between sample means is calculated using the formula:

SE(x1 − x2) =sqrt[(s1^2/n1)+(s2^2/n2)] = sqrt[(18.5^2/18)+(17.8^2/22)] = 4.8862

Therefore, the test statistic Z can be calculated as follows:

Z = [65.3 - 54.5 - 8] / 4.8862= 0.6719

The calculated test statistic (0.67) is less than the critical value (2.33).Thus, we fail to reject the null hypothesis. Therefore, there is not enough evidence to support the alternative hypothesis at a 0.01 level of significance.

Thus, we can conclude that there is insufficient evidence to indicate that the population mean difference is greater than 8. Hence, the null hypothesis is retained.

To learn more about level of significance visit:

brainly.com/question/31070116

#SPJ11

Prove that for every coordinate system ƒ on the line AB, if f(B) < f(A) then a) (AB) = {P∈ AB; f(B) < f(P) < f(A)}
and b) [AB] = {P ∈ AB; f(B) ≤ f(P) ≤ f(A)}

Answers

We have proved both statements a) and b), showing that (AB) = {P ∈ AB; f(B) < f(P) < f(A)} and [AB] = {P ∈ AB; f(B) ≤ f(P) ≤ f(A)}.

To prove the statements a) (AB) = {P ∈ AB; f(B) < f(P) < f(A)} and b) [AB] = {P ∈ AB; f(B) ≤ f(P) ≤ f(A)}, we need to show that the set on the left-hand side is equal to the set on the right-hand side.

a) (AB) = {P ∈ AB; f(B) < f(P) < f(A)}

To prove this statement, we need to show that any point P on the line segment AB that satisfies f(B) < f(P) < f(A) is in the set (AB), and any point on (AB) satisfies f(B) < f(P) < f(A).

First, let's assume that P is a point on the line segment AB such that f(B) < f(P) < f(A). Since P lies on AB, it is in the set (AB). This establishes the inclusion (AB) ⊆ {P ∈ AB; f(B) < f(P) < f(A)}.

Next, let's consider a point P' in the set {P ∈ AB; f(B) < f(P) < f(A)}. Since P' is in the set, it satisfies f(B) < f(P') < f(A). Since P' lies on AB, it is a point in the line segment AB, and therefore, P' is in (AB). This establishes the inclusion {P ∈ AB; f(B) < f(P) < f(A)} ⊆ (AB).

Combining the two inclusions, we can conclude that (AB) = {P ∈ AB; f(B) < f(P) < f(A)}.

b) [AB] = {P ∈ AB; f(B) ≤ f(P) ≤ f(A)}

To prove this statement, we need to show that any point P on the line segment AB that satisfies f(B) ≤ f(P) ≤ f(A) is in the set [AB], and any point on [AB] satisfies f(B) ≤ f(P) ≤ f(A).

First, let's assume that P is a point on the line segment AB such that f(B) ≤ f(P) ≤ f(A). Since P lies on AB, it is in the set [AB]. This establishes the inclusion [AB] ⊆ {P ∈ AB; f(B) ≤ f(P) ≤ f(A)}.

Next, let's consider a point P' in the set {P ∈ AB; f(B) ≤ f(P) ≤ f(A)}. Since P' is in the set, it satisfies f(B) ≤ f(P') ≤ f(A). Since P' lies on AB, it is a point in the line segment AB, and therefore, P' is in [AB]. This establishes the inclusion {P ∈ AB; f(B) ≤ f(P) ≤ f(A)} ⊆ [AB].

Combining the two inclusions, we can conclude that [AB] = {P ∈ AB; f(B) ≤ f(P) ≤ f(A)}.

Therefore, we have proved both statements a) and b), showing that (AB) = {P ∈ AB; f(B) < f(P) < f(A)} and [AB] = {P ∈ AB; f(B) ≤ f(P) ≤ f(A)}.

Learn more about  statement  from

https://brainly.com/question/27839142

#SPJ11


To examine time and sequence, ______ are needed.





curvilinear associations





correlation coefficients





longitudinal correlations





linear statistics

Answers

Longitudinal correlation is a statistical tool used to analyze time and sequence in behavior, development, and health. It assesses the degree of association between variables over time, determining if changes are related or if one variable predicts another. Linear statistics calculate linear relationships, while correlation coefficients measure association. Curvilinear associations study curved relationships.

To examine time and sequence, longitudinal correlations are needed. Longitudinal correlation is a method that assesses the degree of association between two or more variables over time or over a defined period of time. It is used to determine whether changes in one variable are related to changes in another variable or whether one variable can be used to predict changes in another variable over time.

It is an essential statistical tool for studying the dynamic changes of behavior, development, health, and other phenomena that occur over time. A longitudinal study design is used to assess the stability, change, and predictability of phenomena over time. When analyzing longitudinal data, linear statistics, correlation coefficients, and curvilinear associations are commonly used.Linear statistics is a statistical method used to model linear relationships between variables.

It is a method that calculates the relationship between two variables and predicts the value of one variable based on the value of the other variable.

Correlation coefficients measure the degree of association between two or more variables, and it is used to determine whether the variables are related. It ranges from -1 to +1, where -1 indicates a perfect negative correlation, +1 indicates a perfect positive correlation, and 0 indicates no correlation.

Curvilinear associations are used to determine if the relationship between two variables is curvilinear. It is a relationship that is not linear, but rather curved, and it is often represented by a parabola. It is used to study the relationship between two variables when the relationship is not linear.

To know more about Longitudinal correlation Visit:

https://brainly.com/question/6614985

#SPJ11

center (-5,4),When Center (5,4) and tangent to the x axis are given, what is the standard equation of the Circle?

Answers

The given center coordinates are (-5,4), and Center (5,4).The center coordinates of the circle are (5,4), and the radius of the circle is equal to the distance between the center coordinates and the x-axis.

So, the radius of the circle is 4. Now, the standard equation of the circle is (x-a)² + (y-b)² = r²where (a, b) are the coordinates of the center and r is the radius of the circle.We know that the center of the circle is (5, 4) and the radius is 4 units, so we can substitute these values into the equation to get the standard equation of the circle.(x - 5)² + (y - 4)² = 4²= (x - 5)² + (y - 4)² = 16So, the standard equation of the circle is (x - 5)² + (y - 4)² = 16 when the center coordinates are (5, 4) and the circle is tangent to the x-axis.

To know more about coordinates visit:

https://brainly.com/question/32836021

#SPJ11

For the cash flow diagram shown, determine the value of W that will render the equivalent future worth in year 8 equal to $−500 at an interest rate of 10% per year.

Answers

The value of W that will render the equivalent future worth in year 8 equal to $−500 at an interest rate of 10% per year is $-65.22.

Given information

The interest rate per year = 10%

Given future worth in year 8 = -$500

Formula to calculate the equivalent future worth (EFW)

EFW = PW(1+i)^n - AW(P/F,i%,n)

Where PW = present worth

AW = annual worth

i% = interest rate

n = number of years

Using the formula of equivalent future worth

EFW = PW(1+i)^n - AW(P/F,i%,n)...(1)

As the future worth is negative, we will consider the cash flow diagram as the cash flow received.

Therefore, the future worth at year 8 = -$500 will be considered as the present worth at year 8.

Present worth = $-500

Using the formula of present worth

PW = AW(P/A,i%,n)

We can find out the value of AW.

AW = PW/(P/A,i%,n)...(2)

AW = -500/(P/A,10%,8)

AW = -$65.22

Using equation (1)EFW = PW(1+i)^n - AW(P/F,i%,n)

EFW = 0 - [-65.22 (F/P, 10%, 8) - 0 (P/F, 10%, 8)]

EFW = 740.83

Therefore, the value of W that will render the equivalent future worth in year 8 equal to $−500 at an interest rate of 10% per year is $-65.22.

Know more about present worth here,

https://brainly.com/question/31777369

#SPJ11

According to records, the amount of precipitation in a certain city on a November day has a mean of 0.10 inches, with a standard deviation of 0.06 inches.
What is the probability that the mean daily precipitation will be 0.098 inches or less for a random sample of 40 November days (taken over many years)?
Carry your intermediate computations to at least four decimal places. Round your answer to at least three decimal places.

Answers

The probability that the mean daily precipitation will be 0.098 inches or less for a random sample of 40 November days is 0.355.

Step 1: Calculate the standard error of the mean (SEM):

SEM = σ / √n

where σ is the standard deviation and n is the sample size.

In this case, σ = 0.06 inches and n = 40.

SEM = 0.06 / √40

Step 2: Standardize the desired value using the z-score formula:

z = (x - μ) / SEM

where x is the desired value, μ is the mean, and SEM is the standard error of the mean.

In this case, x = 0.098 inches, μ = 0.10 inches, and SEM is calculated in Step 1.

Step 3: Find the cumulative probability associated with the standardized value using a standard normal distribution table or calculator.

P(X ≤ 0.098) = P(Z ≤ z)

where Z is a standard normal random variable.

Step 4: Round the final probability to at least three decimal places.

By following these steps and using the Central Limit Theorem, we can calculate the probability that the mean daily precipitation will be 0.098 inches or less for a random sample of 40 November days. The probability is obtained by standardizing the value using the z-score and finding the cumulative probability associated with it in the standard normal distribution.

To know more about probability, visit:

https://brainly.com/question/18915091

#SPJ11

Simplify (mn)^-6
a. m^6n^6
b.1/m^6n^6
c. m/n^6 d. n/m^6

Answers

The simplified form of (mn)^-6 is 1/m^6n^6, which corresponds to option b.

To simplify the expression (mn)^-6, we can use the rule for negative exponents. The rule states that any term raised to a negative exponent can be rewritten as the reciprocal of the term raised to the positive exponent. Applying this rule to (mn)^-6, we obtain 1/(mn)^6.

To simplify further, we expand the expression inside the parentheses. (mn)^6 can be written as m^6 * n^6. Therefore, we have 1/(m^6 * n^6).

Using the rule for dividing exponents, we can separate the m and n terms in the denominator. This gives us 1/m^6 * 1/n^6, which can be written as 1/m^6n^6.

Hence, the simplified form of (mn)^-6 is 1/m^6n^6. This corresponds to option b: 1/m^6n^6.

To learn more about denominator click here

brainly.com/question/15007690

#SPJ11

A triangle with one angle of 50° could be equilateral. A right-angled triangle could have one of its angles equal to 110°. A triangle with one angle of 50° could be isosceles. An isosceles triangle couldhave one of its angles equal to 110°
A triangle with one angle of 50° could be right-angled

Answers

A triangle with one angle of 50° cannot be right-angled.

In a right-angled triangle, one of the angles is always equal to 90°. Since we are given that one of the angles in this triangle is 50°, the other two angles must add up to 90° (since the sum of all angles in a triangle is always 180°).

In this case, the other two angles would have to add up to 90° - 50° = 40°. However, it is not possible for one of these angles to be 90° and the other to be 40°, as the sum of these angles would be 130°, which is greater than 180° (which is the total sum of all angles in a triangle).

Therefore, a triangle with one angle of 50° cannot be right-angled.

Learn more about triangle  from

https://brainly.com/question/17335144

#SPJ11

Solve the given differential equation: (xtan−1y)dx+(2(1+y2)x2​)dy=0

Answers

The general solution is given by Φ(x, y) + Ψ(x, y) = C, where C is a constant.

To solve the given differential equation:[tex](xtan^{(-1)}y)dx + (2(1+y^2)x^2)dy =[/tex]0, we will use the method of exact differential equations.

The equation is not in the form M(x, y)dx + N(x, y)dy = 0, so we need to check for exactness by verifying if the partial derivatives of M and N are equal:

∂M/∂y =[tex]x(1/y^2)[/tex]≠ N

∂N/∂x =[tex]4x(1+y^2)[/tex] ≠ M

Since the partial derivatives are not equal, we can try to find an integrating factor to transform the equation into an exact differential equation. In this case, the integrating factor is given by the formula:

μ(x) = [tex]e^([/tex]∫(∂N/∂x - ∂M/∂y)/N)dx

Calculating the integrating factor, we have:

μ(x) = e^(∫[tex](4x(1+y^2) - x(1/y^2))/(2(1+y^2)x^2))[/tex]dx

= e^(∫[tex]((4 - 1/y^2)/(2(1+y^2)x))dx[/tex]

= e^([tex]2∫((2 - 1/y^2)/(1+y^2))dx[/tex]

= e^([tex]2tan^{(-1)}y + C)[/tex]

Multiplying the original equation by the integrating factor μ(x), we obtain:

[tex]e^(2tan^{(-1)}y)xtan^{(-1)}ydx + 2e^{(2tan^(-1)y)}x^2dy + 2e^{(2tan^{(-1)}y)}xy^2dy = 0[/tex]

Now, we can rewrite the equation as an exact differential by identifying M and N:

M = [tex]e^{(2tan^{(-1)}y)}xtan^(-1)y[/tex]

N = [tex]2e^{(2tan^(-1)y)}x^2 + 2e^{(2tan^(-1)y)}xy^2[/tex]

To check if the equation is exact, we calculate the partial derivatives:

∂M/∂y = [tex]e^{(2tan^(-1)y)(2x/(1+y^2) + xtan^(-1)y)}[/tex]

∂N/∂x =[tex]4xe^{(2tan^(-1)y) }+ 2ye^(2tan^(-1)y)[/tex]

We can see that ∂M/∂y = ∂N/∂x, which means the equation is exact. Now, we can find the potential function (also known as the general solution) by integrating M with respect to x and N with respect to y:

Φ(x, y) = ∫Mdx = ∫[tex](e^{(2tan^(-1)y})xtan^(-1)y)dx[/tex]

= [tex]x^2tan^(-1)y + C1(y)[/tex]

Ψ(x, y) = ∫Ndy = ∫[tex](2e^{(2tan^(-1)y)}x^2 + 2e^{(2tan^(-1)y)xy^2)dy[/tex]

= [tex]2x^2y + (2/3)x^2y^3 + C2(x)[/tex]

For more such questions on general solution visit:

https://brainly.com/question/30285644

#SPJ8

Given f(x)=5x^2−3x+14, find f′(x) using the limit definition of the derivative. f′(x)=

Answers

the derivative of the given function f(x)=5x²−3x+14 using the limit definition of the derivative is f'(x) = 10x - 3. Limit Definition of Derivative For a function f(x), the derivative of the function with respect to x is given by the formula:

[tex]$$\text{f}'(x)=\lim_{h \to 0} \frac{f(x+h)-f(x)}{h}$$[/tex]

Firstly, we need to find f(x + h) by substituting x+h in the given function f(x). We get:

[tex]$$f(x + h) = 5(x + h)^2 - 3(x + h) + 14$[/tex]

Expanding the given expression of f(x + h), we have:[tex]f(x + h) = 5(x² + 2xh + h²) - 3x - 3h + 14$$[/tex]

Simplifying the above equation, we get[tex]:$$f(x + h) = 5x² + 10xh + 5h² - 3x - 3h + 14$$[/tex]

Now, we have found f(x + h), we can use the limit definition of the derivative formula to find the derivative of the given function, f(x).[tex]$$\begin{aligned}\text{f}'(x) &= \lim_{h \to 0} \frac{f(x+h)-f(x)}{h}\\ &= \lim_{h \to 0} \frac{5x² + 10xh + 5h² - 3x - 3h + 14 - (5x² - 3x + 14)}{h}\\ &= \lim_{h \to 0} \frac{10xh + 5h² - 3h}{h}\\ &= \lim_{h \to 0} 10x + 5h - 3\\ &= 10x - 3\end{aligned}$$[/tex]

Therefore, the derivative of the given function f(x)=5x²−3x+14 using the limit definition of the derivative is f'(x) = 10x - 3.

To know more about derivative visit:

https://brainly.com/question/29144258

#SPJ11

Is an isosceles triangle always right?

Answers

No, an isosceles triangle is not always a right triangle.

Is an isosceles triangle always right?

An isosceles triangle is a triangle that has two sides of equal length and two angles of equal measure. The two equal sides are known as the legs, and the angle opposite the base is known as the vertex angle.

A right triangle, on the other hand, is a triangle that has one right angle (an angle measuring 90 degrees). In a right triangle, the side opposite the right angle is the longest side and is called the hypotenuse.

While it is possible for an isosceles triangle to be a right triangle, it is not a requirement. In an isosceles triangle, the vertex angle can be acute (less than 90 degrees) or obtuse (greater than 90 degrees). Only if the vertex angle of an isosceles triangle measures 90 degrees, then it becomes a right isosceles triangle.

Learn more about isosceles triangles at:

https://brainly.com/question/1475130

#SPJ4

Other Questions
Write a Fortran program that performs Gaussian Elimination and back substitution WITHOUT partial pivotingSetup: Each program will take a single input, the size of the Matrix, N. Your program will allocate and populate the matrix using random numbers. Your program will then start the clock. Run Gaussian Elimination and back subsitution. And then take the stop time. Your program will output the time.Task: Create Gaussian elimination with back substitution.Input: Size of square matrix.Internals: Explicitly or implicitly allocate sufficient memory to a Nx(N+1) floating point Matrixusing a random number generator -- populate the Matrix.Perform Gaussian elimination and back subsitution on the MatrixYour routine should have no output other than the runtime Find a Mbius transformation mapping the unit disc onto the right half-plane and taking z=i to the origin. Given the data stream 11100111. Draw the waveform of the signals using the following encoding schemes:(a) RZ(b) AMI(c) Manchester(d) 2B1Q(e) MLT-3 which factors likely to contribute to subluxation and shoulder pain in hemiplegia? Within the structure of a cell membrane, the phospholipid heads face _____, while the tails face _____. a sample consists of the following data: 7, 11, 12, 18, 20, 22, 43. Using the three standard deviation criterion, the last observation (x=43) would be considered an outliera. trueb. false Define and give the significance of the following, in asubstantial paragraph.Maria Theresa ---> Tennis Court Oath (June20/1789) Given Molecular Formula: C4H8O Draw the lewis structures of all possible constitutional (structural) isomers in the space below. Include all bonds to hydrogens. which of the following characterizes max weber's protestant ethic? truck driver, forklift operatorWhich of these e-commerce job roles requires special training and a license to qualify for the position? Choose two. The procedure BinarySearch (numList, target) correctly implements a binary search algorithmon the list of numbers numList. The procedure returns an index where target occurs in numList,or -1 if target does not occur in numList. Which of the following conditions must be met in order forthe procedure to work as intended?(C) The values in numList must be in sorted order. Cinematic film refers to a film that takes advantage of all the special properties and qualities that make the film medium unique.a.Trueb.False Match the descriptions with the words.presented before the Supreme Court by lawyers representing both sidesinterest groups that file testimonial briefs concerning Supreme Court casesdetermines whether the Supreme Court will hear a case or notwhat the Supreme Court does when it sends a decision back to a lower court with orders to implement itthe principle that courts use to ensure decisions are in line with previous decisionslegal document requesting that an inferior court send records to a higher court Determine if each of the following describes a function with the given domain and codomain. a) f:NN where f(n)=10n. b) f:NZ where f(n)=10n. c) f:NN, where f(n)= nd) h:RR, where h(x)= xe) g:NN, where g(n)= any integer >n Which were included among the "Three Principles of the People"? Select three options. a. communismb. democracy c. socialismd. capitalism d. nationalism americans distrust government _______ people in most wealthy democracies. Exercise 9How many integers between 100 and 999 inclusive1. are divisible by 5?2. are divisible by 4?3. are divisible by 4 and 5?4. are divisible by 4 or 5?5. are divisible by 5 but not 4? 1-Consider the reaction when aqueous solutions of zinc iodide and silver(I) nitrate are combined. The net ionic equation for this reaction is:2-Write a net ionic equation for the reaction that occurs when aqueous solutions of barium hydroxide and hydrofluoric acid are combined. Which characteristic of SMART goals involves setting a start date and a completion date?a. realisticb. time-boundc. measurabled. achievable This is a two part question; the questions are NOT related. a. [5 points] Compare and contrast the Great Depression and the recession due to the Covid-19 Crisis along different dimensions such as: length, severity, impact on economy, policy response, cause, etc. Make sure to expand on your answers and include specific facts (\#s/stats). Do not provide a list of ideas; write out your response in paragraph form. b. [3 points] According to Keynesian Theory, stabilization policy (i.e. Aggregate Demand Management) should not be used to smooth out the business cycle. This is because there are 3 key problems with using stabilization policy (both monetary and fiscal). Briefly discuss 2 of these problems below. Provide complete statements (i.e. in paragraph forms), do not list ideas.